LSAT and Law School Admissions Forum

Get expert LSAT preparation and law school admissions advice from PowerScore Test Preparation.

 Jon Denning
PowerScore Staff
  • PowerScore Staff
  • Posts: 904
  • Joined: Apr 11, 2011
|
#37238
Another Local question in #10, with S immediately before T, creating an ST block. Which MP option does that produce? P in 1 and M in 3. Otherwise we'd have MTP in 5-7 and S couldn't come right before T.

With P 1 and M 3, and ST in a block, we'd have to put either L or O into 2, and either O or L into 4. Why? We can't fit either piece of the ST block in between P and M, and we can't put N into 2 because there's no room for L ahead of it (P is in 1).

So it's one of these:

..... P L M O (ST, N)

..... P O M L (ST, N)

Still some uncertainty, but not much.

This question asks what Cannot Be True, so we need just one answer that causes a problem for the two arrangements above.

Here are all five:

Answer choice (A) is fine, since L can go in 2.

Answer choice (B) is okay, because L can also go in 4.

Answer choice (C) is fine, because the ST block can go in 5 and 6 (in either situation above).

Answer choice (D) is the correct answer. We cannot put T into 5, as that would force S into 4 (from the ST block), and 4 must be either O or L.

Answer choice (E) is allowed because we can have the ST block go in 6 and 7 (which would put N into 5).


Final thought here: if you're running low-ish on time, or worried that you might be by section's end, and you can find the correct answer to these Must or Cannot type questions straight away, it may be worth your while to pick it and move on. That's especially true if you're feeling extremely confident in the game (something that's quite likely by the game's fourth question). Ideally you check them all; realistically efficiency may dictate more drastic measures.

Get the most out of your LSAT Prep Plus subscription.

Analyze and track your performance with our Testing and Analytics Package.